Question

2. Which of the following statements concerning beta is correct? a. A stock with a beta...

2. Which of the following statements concerning beta is correct?

a. A stock with a beta of 0 is expected to provide a rate of return equal to the market portfolio

b. A stock with a beta equal to 1 has no risk

c. Stocks with negative betas have the least amount of risk FALSE

d. A stock with a beta greater than 1 is expected to be more volatile than the market portfolio

Homework Answers

Answer #1

According to CAPM,

market portfolio has beta = 1

Now if any stock has beta = 1, it indicates that if market rises/falls by 1%, stock will also rises/falls by 1%

If beta is greater than 1, the rise/fall in stock will be higher than market

If beta = 0, no market risk, it will provide rate of return equal to risk free asset

If beta is negative, it will follow opposite relation with market. if market rises, stock falls and vice versa

correct statement : d. A stock with a beta greater than 1 is expected to be more volatile than the market portfolio

[[[]]]

Know the answer?
Your Answer:

Post as a guest

Your Name:

What's your source?

Earn Coins

Coins can be redeemed for fabulous gifts.

Not the answer you're looking for?
Ask your own homework help question
Similar Questions
Which of the following statements is CORRECT? Select one: a. The beta of a portfolio of...
Which of the following statements is CORRECT? Select one: a. The beta of a portfolio of stocks is always smaller than the betas of any of the individual stocks. b. The beta of a portfolio of stocks is always larger than the betas of any of the individual stocks. c. It is theoretically possible for a stock to have a beta of 1.0. If a stock did have a beta of 1.0, then, at least in theory, its required rate...
Which of the following statements is most correct? The required rate of return of a diversified...
Which of the following statements is most correct? The required rate of return of a diversified portfolio with Beta of 1 is typically greater than the Market Risk Premium. A stock with a negrative beta must have a negative required rate of return. If a stock's beta doubles its required rate of return must double. If a stock has a beta equal to 1.0, its required rate of return will be unaffected by changes in the market risk premium. None...
Stock A's beta is 1.5 and Stock B's beta is 0.5. Which of the following statements...
Stock A's beta is 1.5 and Stock B's beta is 0.5. Which of the following statements must be true, assuming the CAPM is correct. a. Stock A would be a more desirable addition to a portfolio then Stock B. b. Stock B would be a more desirable addition to a portfolio than A. c. When held in isolation, Stock A has more risk than Stock B. d. In equilibrium, the expected return on Stock B will be greater than that...
Stock A's beta is 1.5 and Stock B's beta is 0.5. Which of the following statements...
Stock A's beta is 1.5 and Stock B's beta is 0.5. Which of the following statements must be true, assuming the CAPM is correct. a. Stock A would be a more desirable addition to a portfolio then Stock B. b. In equilibrium, the expected return on Stock B will be greater than that on Stock A. c. When held in isolation, Stock A has more risk than Stock B. d. Stock B would be a more desirable addition to a...
Which of the following statements concerning the variance are correct? (note: there can be more than...
Which of the following statements concerning the variance are correct? (note: there can be more than one correct answer) Group of answer choices a.The larger the variance, the greater the total risk of the investment. b.If a stock portfolio is well diversified, then the portfolio variance may be less than the variance of the least risky stock in the portfolio. c.The larger the variance, the smaller the standard deviation. d.The larger the variance, the more the actual returns tend to...
Stock X has a beta of 0.5 and Stock Y has a beta of 1.5. Which...
Stock X has a beta of 0.5 and Stock Y has a beta of 1.5. Which of the following statements is most correct? Select one: a. If expected inflation increases (but the market risk premium is unchanged), the required returns on the two stocks will decrease by the same amount. b. If investors' aversion to risk decreases (assume the risk-free rate unchanged), Stock X will have a larger decline in its required return than will stock Y. c. If you...
1. Asset 1 has a beta of 1.2 and Asset 2 has a beta of 0.6....
1. Asset 1 has a beta of 1.2 and Asset 2 has a beta of 0.6. Which of the following statements is correct? A. Asset 1 is more volatile than Asset 2. B. Asset 1 has a higher expected return than Asset 2. C. In a regression with individual asset’s return as the dependent variable and the market’s return as the independent variable, the R-squared value is higher for Asset 1 than it is for Asset 2. D. All of...
Which of the following statements is true with respect to beta? Multiple Choice A - All...
Which of the following statements is true with respect to beta? Multiple Choice A - All of the above. B-- A stock with a beta > 1 is more volatile than the market portfolio. C -- The market portfolio has a beta of "0". D -- A stock with a beta < 1 will outperform the market portfolio when the market is up.
QUESTION 17 Which of the following statements is most correct? a. An increase in expected inflation...
QUESTION 17 Which of the following statements is most correct? a. An increase in expected inflation could be expected to increase the required return on a riskless asset and on an average stock by the same amount, other things held constant. b. A graph of the SML would show required rates of return on the vertical axis and standard deviations of returns on the horizontal axis. c. If two "normal" or "typical" stocks were combined to form a 2-stock portfolio,...
Which of the following statements about the beta coefficient is false? A A stock’s beta coefficient...
Which of the following statements about the beta coefficient is false? A A stock’s beta coefficient measures its volatility relative to the market portfolio. B A stock’s beta coefficient can be estimated by plotting the stock’s returns versus the market portfolio’s returns. C A stock’s reported beta coefficient is based on forecasted future volatility. D A stock with a beta coefficient greater than 1.0 is said to be riskier than the market portfolio. E Using the capital asset pricing model,...